This is a weakening question, as the stem asks us: Which one of the following, if true, argues most strongly against the explanation reported in the passage?
The first sentence of the stimulus should be ringing the phenomenon bell in your head; the songbird population of England has decreased within recent years. The following sentence gives us the explanation hinted at in the question stem, a hypothesis for why the decrease occurred. This explanation is that the songbird decrease is correlated with the increase in population of one of the songbird’s predators, magpies, who specifically target their young and eggs. This is a classic weakening question format of phenomenon-correlation-causal hypothesis, a correct answer will likely either present an alternate hypothesis or undermines the correlation. Let’s see what we end up with:
Answer Choice (A) The stimulus specifically states that the phenomenon has occurred in recent years, so this is irrelevant.
Answer Choice (B) This answer connects to the fact that magpies target songbird eggs, but you should recognize that it gives us very little information to work with.
Correct Answer Choice (C) This answer does one of the things we predicted, it undermines the correlation between songbird population decreases and magpie population increases which the explanation inferred as a causal relation. If most cases where songbird populations decreased involved no change in the magpie population, then magpie population growth isn’t an appealing explanation for the songbird decrease.
Answer Choice (D) This answer gives us an explanation of why the magpie population has increased, but we are interested in weakening the casual hypothesis that the magpie population increase caused the songbird population decrease.
Answer Choice (E) I’m glad the magpies are getting a well-rounded diet, but even if magpies eat other stuff, the argument can still infer that more magpies means more eaten songbird eggs and babies.
This is a Resolve, Reconcile, and Explain Question. We know this because of the question stem, which includes: “Most helps to resolve the apparent discrepancy?”
RRE questions will require an explanation of a conflicting set of facts. Our correct answer choice, when plugged back into the stimulus, will resolve the discrepancy by explaining how the two sides of the apparent conflicting issues actually make sense together. The correct answer will use both sides, though not necessarily explicitly, to explain the conflict. Often, the test will entice you to make naive assumptions about the conflict - don’t fall for it! Your approach should fall under the “this seems wrong because of xyz, but I can think of a few reasons it could work.”
Our first sentence is pretty long; however, if you break it down by figuring out the subject and then building onto that, that could be helpful. We're told the aquifers themselves don’t have bacteria but water is chlorinated because it could be contaminated due to cross-contamination with old pipes. Then we’re given a specific instance of municipalities getting water from the same aquifer: there are 50 of them, 30 chlorinate their water and 20 do not. The 20 that do not chlorinate their water have lower levels of bacteria in their water when compared with the 30 that do chlorinate their water. This is surprising! Wouldn't chlorinating the water help reduce the bacteria, especially compared to those who do not chlorinate the water?
The discrepancy is when these 50 municipalities are getting water from the same aquifer, how is it that those who chlorinate their water, supposedly killing bacteria, have higher levels of bacteria than those who do not chlorinate their water? This is a legitimate conflict, but there are things that could explain this. For example, what if the pipes for those 30 municipalities are old and gross and that’s why they chlorinate their water? That could potentially explain it.
Let’s look at the answer choices:
Answer Choice (A) How does this help account for the unexpected level of bacteria in chlorine water? It doesn’t! This information isn’t helpful for our issue.
Answer Choice (B) This could be a reason why the 20 did not chlorinate their water; however, does this explains why the 30 that did have higher levels of bacteria? No. This is out.
Answer Choice (C) This adds more information that deepens the conflict - not only did they not add chlorine, but there are also no other chemicals that they added to disinfect their water. If the opposite was stated, it could work. But as is, this is out.
Answer Choice (D) This answer choice notes something about these two groups that were similar, not how they were different. This won't help us resolve any conflict. This isn’t good.
Correct Answer Choice (E) This answer choice says that the pipes for the 20 municipalities are cleaner because their government has strict requirements for them, and therefore they would not need to add chlorine because their pipes are already in good shape compared to those municipalities that have to chlorinate their water.
This is a Resolve, Reconcile, and Explain Question. We know this because of the question stem: “Most helps to resolve the apparent discrepancy?”
RRE questions will require an explanation of a conflicting set of facts. Our correct answer choice, when plugged back into the stimulus, will resolve the discrepancy by explaining how the two sides of the apparent conflicting issues actually make sense together. The correct answer will use both sides, though not necessarily explicitly, to explain the conflict. Often, the test will entice you to make naive assumptions about the conflict - don’t fall for it! Your approach should fall under the “this seems wrong because of XYZ, but I can think of a few reasons it could work.”
The first sentence reports on what the city’s CDC says about rabid raccoons: it’s more serious now than it was two years ago because of a percentage increase in raccoons with rabies. However, the second sentence gives us an interesting statistic: the number of confirmed cases has gone down from two years ago to this year.
This plays off a very important distinction, which often comes up in flaw-type questions. This stimulus is comparing percentages to numbers. There is a whole host of reasons based on this alone that could explain this discrepancy. Other reasons could be that the data doesn’t accurately reflect the actual confirmed cases of rabid raccoons.
Answer Choice (A) Other animals? Does this explain the discrepancy between the percent and numbers? Nope. This is out.
Correct Answer Choice (B) With a significant portion of raccoons succumbing to rabies, this could explain the apparent discrepancy between numbers and percentages. Let's say two years ago there were 130 rabid raccoons and <25% had rabies, so 32 raccoons. Now, since there is a substantial decrease, it's possible that the raccoon population is 34, and 18 of them have rabies. 18 is still over 50% of 34 - this would explain the numbers! If you’re thinking “There are fewer of them in general, so how is this a serious problem??” However, a greater proportion of raccoons having rabies is still a pretty serious problem.
Answer Choice (C) This would be correct if we are assuming that the cases of rabies counted a few years ago were confused with distemper, and so the actual number of confirmed cases two years ago is a lot lower. However, that’s a pretty big assumption, one we cannot make. Without this assumption, the answer isn’t good.
Answer Choice (D) Outside information is okay, but saying that these animals are nocturnal does not help explain the conflict here.
Answer Choice (E) The adds more information to the facts but does not help resolve. This is out.
This is a most strongly supported question, as the question stem asks: The statements above, if true, most strongly support which one of the following conclusions?
Our stimulus begins with a description of rheumatoid arthritis, specifically the mechanism by which it occurs. Immune system misfunction leads to attacks on joint cells which causes the release of a hormone which in turn causes pain and swelling. So far all we have is a causal chain; immune misfunction > join cells attacked > hormone released > pain and swelling. The next sentence gives us a bit more information on this hormone; it is normally only triggered by injury and infection. Since we know the hormone causes pain and swelling, what we should infer from this is that injury or infection can also lead to the same pain and swelling as rheumatoid arthritis. Finally, we are told about a new medication that includes a protein that blocks the pain and swelling hormone. Let’s see what the correct answer infers from this:
Answer Choice A We are only told that the medication “contains” a hormone-inhibiting protein for a hormone that specifically causes pain and swelling; nothing in the stimulus mentions or suggests that it also repairs cells. This answer is consistent with the stimulus, but completely unsupported.
Answer Choice B This is consistent with what we know but not something any of the information we have been given would support. We are told nothing about how the medications benefits compares to any possible harmful side effects.
Correct Answer Choice (C) This is where the inference we made from the second sentence comes in handy. We know that the hormone can be caused by injury and we know that it causes pain; if the medication stops this pain that would result from an injury, then we have reason to believe that at least some of the pain that results from a join injury wouldn’t occur because of the medication. C is the only answer that has any significant support in the stimulus.
Answer Choice (D) We have been told nothing about the mechanisms by which other immune system disorders cause pain. We are not entitled to make the assumption that they are similar enough to rheumatoid arthritis that the medication could be adapted, and therefore this answer has little support.
Answer Choice (E) Similar to D, we have been given no information about other joint diseases. Maybe they do all involve the same hormone.
This is a most strongly supported question, as the question stem asks: If the statements above are true, which one of the following is most strongly supported by them?
Our stimulus begins by telling us that in the last quarter century the introduction of labor-saving tech has greatly reduced the average amount of time required for a worker to produce a corresponding output. This change has the potential to reduce the number of hours workers spend working every work, and increasing the amount of time they have to themselves. Who doesn’t like the sound of that! Unfortunately, it seems the average amount of leisure time has increased at only half the rate that output has grown. What gives! Since this is a MSS question, we should treat all this information as a series of premises. The correct answer will be the conclusion which can follow from these premises with the least assumptions required. Let’s see what are options are like:
Answer Choice (A) This conclusion requires a lot of assumptions. More specifically, we have been told nothing about what workers are spending their money on. Even if they have more leisure time total, what if leisure activities have gotten much cheaper?
Answer Choice (B) We have only been told about what these technologies have done to worker output and leisure time, we know nothing about the amount of jobs they have created or replaced.
Answer Choice (C) Again, this just isn’t something we know much about. For this to follow from the information in the stimulus, we have to make a lot of assumptions.
Answer Choice (D) We’ve been told nothing about what was anticipated compared to the increase that actually occurred.
Correct Answer Choice (E) If the average hourly output has increased, and at double the rate at which average leisure time has increased, then it is highly likely that the average weekly output is higher than it was before the introduction of labor-saving technology. The weekly output is determined by the amount of hours worked in a week and the hourly output, and we know that hours worked per week has declined (leisure time) less than hourly output has increased.
This is a sufficient assumption question because the question stem says: “conclusion is properly drawn… which one is assumed?”
Sufficient assumption questions tend to be very formal. We’re looking for a rule that would 100% validate the conclusion, specifically by bridging the premise and conclusion through the rule. Not only are we extrapolating the rule from our argument, but we’re also using that rule to render the argument “valid.” The way to prephrase our answer choice is by tying our premises and conclusion together into a rule: “If [premise] → then [conclusion].” Sometimes though, rules are a little too chunky and don’t capture the gap accurately. This question is a great example of why.
The first sentence is pretty straightforward: photovoltaic power plants (PVP) create electricity through sunlight. We’re also told that it’s cheaper than it was 20 years ago and included the costs of construction and operation in this statement. The passage then notes that corresponding costs have increased for traditional power plants (TP). Okay, all good so far – the author is noting the change of costs for each separately.
The conclusion is that PVP is less expensive than TP. Wait – how can we draw a comparison between the two? We only know what’s going on with their respective costs. We can’t make a comparison between the two when we only know what’s going on within the two respectively. Let’s walk through this:
Imagine in 2003 the cost for PVP was $500. Since then, it’s decreased by 1/10, so the cost today is $50.
Now, for TP, in order for our conclusion to be true, the cost of TP in 2023 would have to be above PVP’s costs in 2023. So, something like: in 2003, the cost for TP was $600, and now the cost is $6000. (A note that “increasing” is a very vague term on the LSAT – we can’t really tell by how much something increased.)
But – does that have to be the case? NO!! It’s totally possible that the cost for TP was $20 in 2003, and then increased to $45. This goes against the conclusion. The problem here is that the premises are only giving information for comparison within the group, and our conclusion is about what’s going on between the groups. There is a mismatch.
If you’re having trouble seeing the comparison issue, let’s take another context. Let’s use the LSAT score as an example. What this argument is saying is the following... I took the LSAT twice and my score went up between those two takes. My friend also took the LSAT but her score went down from her first take. Therefore, I score higher on my LSAT than my friend did.
Is this conclusion valid? Not really - it could be that I went from a 150 to a 154; but my friend went from a 160 to a 159. This is the issue: having information about what’s going on with me and separately with my friend doesn’t allow for a conclusion comparing my score with my friend’s score.
Back to our argument, to close the gap here, a rule feels a little complicated. We need something explicit to address this gap. Something that draws a comparison between the two groups, not just within them. Something like, even 20 years ago, the cost for PVP was way lower than TP plants.
Answer Choice (A) This answer choice is just a repetition of what was stated in the stimulus above. A repetition of our stimulus isn’t going to help us out here.
Answer Choice (B) This isn’t correct – we don’t care about electricity produced 20 years ago. The technology could have improved and PVP produces much more; on top of this, the conclusion is about the cost of electricity now.
Answer Choice (C) Plugging this back into our stimulus will show how this is completely useless to our argument. This answer choice is basically saying that PVP and TP technology works in different ways; so, what? It’s possible to have two ways of doing something and both of them are great. This doesn’t make our conclusion valid.
Correct Answer Choice (D) We’re saying that even back then, PVP was 10 times less than TP. If PVP costs went down by a tenth and we also know that TP costs have increased, then we can properly draw our conclusion.
Answer Choice (E) Once again, we have more information about what’s going in within the groups and no information on what these costs are relative to each other.
This is a strengthening question, which we know because the question stem asks: Which one of the following, if true, most strongly supports the conclusion drawn by the entomologists?
The stimulus tells us that a few years ago some entomologists (insect scientists) introduced a fungus to oak forests to get rid of gypsy moths which eat oak leaves. The fungus is poisonous specifically to the caterpillars of these moths, and in the years since the population of both the caterpillars and moths has declined where the fungus was introduced. The entomologists conclude with a hypothesis that it was the fungus which caused the moth decline.
Like many strengthening questions this one involves a causal hypothesis; the passage gives us a correlation between the introduction of fungus and the decline of the moths and caterpillars, and we are supposed to find the answer which provides more justification for the entomologists’ hypothesis that this correlation really is a causal relation. With that in mind, let’s go about eliminating some of these answers:
Correct Answer Choice (A) This answer gives us a subset of gypsy moths with which to compare the effects on the other susceptible moths. The only difference between the rest of the gypsy moths and this strain which we are told about is that the subset is immune to the poisonous fungus. This makes them a really good control population to test our hypothesis. Since the only known factor by which the two groups differs is whether they are affected by the fungus, and the resistant group is growing in size while the moths as a whole decline, this is good evidence that it is the fungus which has caused the overall decline of the moths and caterpillars.
Answer Choice (B) This question introduces other insects to the equation, and you should immediately recognize that no mention is made of them in the entomologists’ hypothesis. Other insect groups are irrelevant to the phenomenon of gypsy moth decline.
Answer Choice (C) This answer introduces another possible cause of the gypsy moth decline, predation. Considering we are trying to support the fungus hypothesis, this answer is if anything weakens what we want to strengthen, and an increase of moths and caterpillars following a drop in predation does nothing to explain the overall significant decline which we are interested in.
Answer Choice (D) This answer introduces an entirely new phenomenon, a decline in oak tree populations caused by air pollution and acid rain. This might be a good answer if this were a weakening question, since the moths feed on oak trees and therefore the decline in oak trees would be an alternative explanation for the decline of moths. But since this is a strengthening question this answer does the opposite of what we are looking for, and is consequently incorrect.
Answer Choice (E) Just as A introduced a control population which was immune to the fungus, E introduces a control population where the fungus has not been introduced. The problem is that where A suggested the fungus as a cause because relative to the general moth population the fungus-immune moths seemed to be doing well, here the moths seem to be declining regardless of whether the fungus is present. This shows that the correlation between population and fungus decline is very weak, and therefore weakens the fungus hypothesis which we want to strengthen.
This is a strengthening question, as the stem asks us: Which one of the following, if true, most strongly supports Ping’s conclusion?
The passage takes the form of a dialogue between Winston and Ping over a transit authority’s attempt to operate without a budget deficit. We know from the question stem that we are interested in Ping’s conclusion. The first sentence appears a bit more complicated than it really is; it throws a lot of information and negations at us but what should really jump out at you is the “unless”. If we convert this unless into a conditional, we get something along the lines of “if the PTA does not cut late night services, then it cannot fulfill its mandate.” Whenever you see a sentence in an argument begin with “since” and then transition after a comma into a second clause, you should pick up the hint that the author is about to give a conclusion after introducing a premise. “Since [premise] is true, [conclusion].” Winston does exactly this, and gives us the conclusion of his argument that the cuts would reduce the deficit and should be made. As a cherry on top, he ends with another premise telling us that fare increases take two years, eliminating another possible remedy at least in the immediate future.
Ping responds to Winston by pointing out that cutting late-night service would not only affect fares purchased during that period, but would also decrease fares during the day, insofar as there are “those riders” who do round trips with one ride in the day and one during the late-night period. It makes sense when you think about it, if you work a job that finishes late, you might still commute to work during the day. If you can’t take the transit home after work at night, then you are more likely to find another way to commute during the day as well. It is a bit difficult to identify the conclusion in this argument, because it actually includes two. It is tempting to consider the last sentence the conclusion that the question stem told us about, because it begins with the indicator “thus”. However, this is actually a sub-conclusion which supports Ping’s overall position which he states in his opening sentence; the proposed service cuts might cost more than they would save. The correct answer will support Ping’s position that cutting late night service could lose the PTA money. Let’s look at the answers:
Correct Answer Choice (A) What this answer does is give us a bit more insight into those riders that Ping mentions. Ping’s argument as stated assumes those riders represent a significant enough proportion of ridership that the loss of their fares might be greater than the savings of cutting late-night services. For all we know, these riders could be a very small group, or not even exist. What A does is fill in this gap in the argument, by explicitly telling us that those riders make up almost a quarter of all PTA round trips. While 23% isn’t as strong support as a higher percentage, it’s important not to let this throw you off; the proof that those riders exist at all adds support to Ping’s conclusion.
Answer Choice (B) This answer tells us the riders’ opinion on cutting late-night services. Unfortunately, we don’t care what the riders think. Ping’s conclusion is simply a prediction that if the cuts happen, an undesirable result (loss of money) might occur. This is true regardless of the opinions of riders.
Answer Choice (C) C like B introduces completely irrelevant information to Ping’s conclusion. Winston mentions that it is a long process to get a fare increase approved, and all this does is strengthen that a fare increase is not a viable solution for the PTA to avoid a budget deficit. But fare increases have no bearing on Ping’s argument because it is a prediction about late night service cuts.
Answer Choice (D) Whichever LSAC writer did this question sure loves percentages, and we get another one thrown at us. Once again this answer is irrelevant to Ping’s argument. The relative severity of the deficit does not affect Ping’s claim about the potential for service cuts to lose the PTA money.
Answer Choice (E) This answer doesn’t give us enough information, and if anything hurts Ping’s argument by introducing another cost associated with late-night services which might outweigh the fares earned from those riders. While it tells us that the cash bonuses are “significant”, we don’t know how many bus drivers there are or even whether the late-night service is composed of mostly buses as opposed to trains, streetcars, or what.
Here we have a weakening question, as the stem asks: Which one of the following, if true, most weakens the patent reviewers’ argument?
Our stimulus begins with some context about a patent application for a genetically engineered lab mouse that got denied. In support of their decision, the patent reviewers argued that the mouse was a new variety of animal and they aren’t allowed to approve patents for new animal varieties. This argument is a case of rule application, a conditional rule (new animal variety → no patent) is used to justify a particular conclusion (no patent for the lab mouse). It is important to note that we have not been given an explanation of what makes an animal a new variety. We know the mouse has been genetically engineered, but not whether this means it really is a new variety. That the mouse is a new variety is an assumption upon which the reviewers’ argument depends. Since there are many ways to weaken an argument, we can’t assume that this assumption will play a role in the correct answer, but we should take note of it regardless. Let’s look at the answer choices.
Correct Answer Choice (A) This answer choice completely undermines the reviewers’ argument. If the new variety rule only applies to farm animals, than it can’t be used to support denying the laboratory mouse patent.
Answer Choice (B) Who cares? This answer choice is completely irrelevant to the patent reviewers’ decision and reasoning.
Answer Choice (C) All this tells us is that the patent reviewers are consistent in their application of the rule. It does nothing to undermine their reasoning for why the rule supports denying the lab mouse patent.
Answer Choice (D) The rule that the reviewers’ cited as their reason specifically applies to animal varieties. It is entirely consistent with their reasoning that they would approve patents which don’t violate the rule. D is incorrect.
Answer Choice (E) Similar to C this just gives us some extra context that the patent reviewers have consistently applied the rule to past animal varieties. What we want is a reason why the rule shouldn’t apply to the lab mouse, which A gives us.
This is a most strongly supported question, as it asks: The statements above provide the most support for which one of the following?
The stimulus begins by informing us that there is a correlation between the distance animals travel and the size of their groups, and their diets. This just means what an animal eats is consistently related to how far it travels and what kind of group it lives with. So maybe herd animals that travel long distances usually live off mostly grass or something; we don’t get any details, just that the relationship exists. Next we learn that diet itself correlates with the animals’ faces and teeth. And that’s it! With a short MSS stimulus like this, we should be thinking about what kind of inferences we can make with so little information. In this case, if there is a connection between traveling/group behavior and diet, and between diet and face/teeth, then maybe you can somewhat reliably predict how an animal travels/groups just based off what kind of face/teeth it has. Let’s take a look at the answer choices:
Answer Choice (A) We’ve been told that diet and travel correlate, but not how. This answer requires a lot of assumptions about the actual details of the correlation.
Answer Choice (B) We’ve been told that diet correlates with face and teeth shape and size, but not the details of this correlation and certainly nothing about how overall size correlates with diet.
Answer Choice (C) This is a very specific detail. To infer it just off the two sentences we were given would require a ton of assumptions.
Answer Choice (D) What should really signal that this answer is wrong is “all that is needed”. That is a very strong claim to draw from our two sentences.
Correct Answer Choice (E) Compare the “all that is needed” of D to the mere “can” of this answer choice. Exactly as we predicted in our pre-phrase, this answer makes the relatively small inference that if diet correlates with teeth and face, and diet also correlates with how an animal travels, then there might be a correlation between teeth and face, and how an animal travels.